79
Test #20 Q.Id = 603 Question 1 Of 26 Item Review Division : Cardiology 36-year-old male patient, who has a history of Marfan's syndrome, presents with sudden onset of severe central tearing chest pain radiating to his back. The pain is 9/10 in severity and has no relation with exertion. He is a non- smoker and non-alcoholic. Measurement of his BP shows a difference of 35 mmHg between his two arms. Chest auscultation shows the presence of mid-systolic click and there is no murmur. What is the next most appropriate diagnostic step in the management of this patient? A . Transesophageal echocardiography B . MRI of the chest C . Cardiac enzymes D . Coronary angiogram E . Ventilation-perfusion scan F . Arterial blood gas analysis Explanation: This patient is most likely suffering from acute aortic dissection for which a transesophageal echocardiography or computed tomography of the chest are the diagnostic studies of choice. Marfan's syndrome predisposes to the development of aortic dissection. Tearing pain with radiation to the back and a difference in BP of greater than 30 mmHg between two arms are important clinical clues for aortic dissection. MRI is used to establish the diagnosis of aortic dissection when its presentation is chronic or when there is no hemodynamic instability.

T 20 Cardiology

Embed Size (px)

Citation preview

Page 1: T 20 Cardiology

Test #20   Q.Id = 603 Question 1 Of 26Item Review

Division : Cardiology

36-year-old male patient, who has a history of Marfan's syndrome, presents with sudden onset of severe central tearing chest pain radiating to his back. The pain is 9/10 in severity and has no relation with exertion. He is a non-smoker and non-alcoholic. Measurement of his BP shows a difference of 35 mmHg between his two arms. Chest auscultation shows the presence of mid-systolic click and there is no murmur. What is the next most appropriate diagnostic step in the management of this patient?

A. Transesophageal echocardiography  B. MRI of the chest  C. Cardiac enzymes  D. Coronary angiogram  E. Ventilation-perfusion scan  F. Arterial blood gas analysis

Explanation:

This patient is most likely suffering from acute aortic dissection for which a transesophageal echocardiography or computed tomography of the chest are the diagnostic studies of choice. Marfan's syndrome predisposes to the development of aortic dissection. Tearing pain with radiation to the back and a difference in BP of greater than 30 mmHg between two arms are important clinical clues for aortic dissection.

MRI is used to establish the diagnosis of aortic dissection when its presentation is chronic or when there is no hemodynamic instability.

EKG and cardiac enzymes are of great importance when coronary artery disease is suspected. This patient has no risk factors for CAD and his pain is not typical of CAD.

A ventilation perfusion scan is indicated when pulmonary embolism is suspected. This patient's clinical picture is not compatible with the diagnosis of pulmonary embolism. He has no risk factors for DVT.

ABG is not an issue here.

Educational Objective: Transesophageal echocardiography or computed tomography are the diagnostic studies of choice for suspected aortic dissection.

57% of people answered this question correctly

Test #20   Q.Id = 614 Question 2 Of 26Item Review

Division : Cardiology  

Page 2: T 20 Cardiology

A 38-year-old Asian immigrant presents for the evaluation of exertional dyspnea with minimal activity. His past medical history is significant for acute rheumatic fever. He denies any fever, chest pain, cough, malaise or weight loss. His PR: 70/min and regular; BP: 126/76mmHg; Temperature: 37.2C(99F). His apex beat is tapping and non-displaced. Auscultation of his lungs shows crepitations in both lower lung fields. Auscultation of heart reveals a loud first heart sound, mid-diastolic rumbling and a low-pitched murmur at the apex with an opening snap. The murmur is accentuated by mild exercise. Chest X-ray shows straightening of the left border of the heart and presence of Kerley B lines. EKG shows left atrial enlargement. Which of the following findings is a hallmark of the suspected disease?

  A. Left atrial enlargementB. Elevated left atrioventricular pressure gradient

  C. Wide pulse pressure  D. Elevated left ventricular diastolic pressure  E. Atrial fibrillation

Explanation:

The hallmark finding of MS is elevated left atrioventricular pressure gradient that ultimately leads to left atrial enlargement. The elevated left atrial pressures are transmitted to the pulmonary veins, capillaries, and arteries. These phenomena are responsible for the exertional dyspnea and later, pulmonary hypertension and right heart failure. Enlargement of the left atrium predisposes to the development of atrial fibrillation which is quite common in mitral stenosis but is not the hallmark finding.

The left ventricular diastolic pressure is normal in pure mitral stenosis. These pressures become elevated when there is coexistent mitral regurgitation, aortic valve disease, systemic hypertension, or coronary artery disease.

A wide pulse pressure is not a feature of mitral stenosis. It occurs in aortic insufficiency and other conditions in which circulation is hyperdynamic.

Educational Objective: The hallmark finding of MS is elevated left atrioventricular pressure gradient.

23% of people answered this question correctly

Test #20   Q.Id = 631 Question 3 Of 26Item Review

Division : Cardiology  

A 60-year-old male presents to the emergency room with the chief complaint of progressive dyspnea and fatigue. He denies any chest pain, syncope, cough, or edema. He suffered an acute anterior wall MI 1 month ago. He does not have a history of COPD or asthma. He is taking aspirin, metoprolol, enalapril, and simvastatin. Chest auscultation reveals bilateral crackles in his lower chest.

Page 3: T 20 Cardiology

Examination of the precordium reveals a double apical impulse. Cardiac auscultation reveals the presence of an S3 and a pansystolic murmur at the mitral area that radiates to the apex. Chest x-ray shows prominence of the left heart border. EKG shows previously present unchanged Q waves and a persistent ST segment elevation in the anterior leads. Based on these findings, what is the most likely underlying cause of his symptoms?

  A. Ventricular septal defect  B. Papillary muscle rupture  C. Free ventricular wall rupture  D. Rupture of chordae tendinae

E. Left ventricular aneurysmExplanation:

This patient most likely has a left ventricular aneurysm, which is a late complication of an acute MI. It usually occurs after an anterior wall MI and may be either asymptomatic, or it may present with congestive heart failure, an arterial embolism, or sustained ventricular arrhythmias. Precordial examination may show a double apical beat. Auscultation of the heart may reveal an additional S3 or S4 and sometimes a murmur of mitral regurgitation is present due to papillary muscle dysfunction, annular dilatation, or abnormal left ventricular geometry. Chest x-ray usually shows a characteristic prominence of the left border of the heart. The EKG shows persistent ST segment elevation.

Ventricular septal defects and papillary muscle rupture may complicate an acute MI. However, these complications occur much earlier within the first week post MI and they usually present acutely with a new pansystolic murmur and cardiogenic shock.

Rupture of chordae tendinae is not a complication of MI and it usually occurs secondary to trauma or infective endocarditis.

Free-wall rupture is an early mechanical complication of MI. It does not produce a murmur and it presents with cardiogenic shock.

Educational Objective:LV aneurysm can cause CHF in a patient who sustained an anterior wall MI in the past. A double apical beat and persistent elevation of the ST segment are important diagnostic clues.

21% of people answered this question correctly

Test #20   Q.Id = 640 Question 4 Of 26Item Review

Division : Cardiology  

A 66-year-old white male comes for the evaluation of his progressively

Page 4: T 20 Cardiology

worsening dyspnea and bilateral ankle swelling. He complains of dyspnea at rest, orthopnea, and paroxysmal nocturnal dyspnea. He has a history of HTN for 15 years and suffered three MIs in the past. He was treated with a 2 vessel CABG 6 years ago and his repeat catheterization 1 year ago showed partial occlusion of the grafts and diffuse native vessels disease. It was decided that he is not a candidate for any intervention or surgical procedure. His ejection fraction is 30%. His medications include aspirin, simvastatin, isosorbide, and metoprolol. On examination, his PR: 86/min; BP: 110/70 mmHg; RR: 14/min; Temperature: 37.2C(99F). His neck veins are distended and bilateral pitting ankle edema is present. Auscultation of his lungs is significant for the presence of bilateral crepitations; he also has an S3 gallop without any murmurs. Chest x-ray shows enlargement of cardiac silhouette and bilateral pulmonary edema. Addition of which of the following medication to his regimen will increase this patient's overall long-term survival?

A. ACE inhibitor  B. Calcium channel blockers  C. Loop diuretics  D. Digoxin  E. Dobutamine  F. Home oxygen therapy

Explanation:

Of all the above-mentioned drugs only ACE inhibitors have been shown to decrease long-term mortality in patients with ischemic cardiomyopathy and LV systolic dysfunction. Several randomized and meta analysis studies have proved this benefit. ACE inhibitors should be considered in all patients with coronary artery disease who have an EF of less than 40%.

Therapy with digoxin does not improve survival but it provides symptomatic benefit in patients with systolic dysfunction and sinus rhythm. They are also used to control ventricular rate in patients of heart failure who have atrial fibrillation. Use of digoxin has been associated with less hospitalizations and lower health care costs.

Loop diuretics are very useful in providing symptomatic relief in those patients with heart failure who have pulmonary or peripheral edema. However, they have not been shown to improve survival. The only diuretic, which has shown some survival benefit in patients with severe heart failure, is spironolactone.

Calcium channel blockers have no role in the treatment of CHF and they do not provide any survival benefit. In fact, they can worsen the peripheral edema by causing peripheral venous dilatation.

Dobutamine is used for inpatient treatment of heart failure when it is severely decompensated and it does not provide a survival benefit.

Page 5: T 20 Cardiology

Home oxygen therapy has been shown to have a survival advantage in patients who have COPD and right-sided heart failure from pulmonary hypertension. There is no role for oxygen in left sided heart failure since the problem is not in the lung.

Educational Objective:Know that ACE inhibitors are the survival improving drugs in congestive heart failure.

52% of people answered this question correctly

Test #20   Q.Id = 1704 Question 5 Of 26Item Review

Division : Cardiology

A 67-year-old male is presented to ER because of chest pain. He has a history of stable angina and has been on aspirin and isosorbide. His other medical problems include HTN and bronchial asthma. Occasionally, he takes albuterol inhalations. He is started on a nitroglycerine drip. He feels little lightheaded. His initial EKG is shown below. His vital signs are, BP: 100/60 mm Hg, RR: 16/min and Temperature: 37C(98F). Physical examination reveals clear chest. The next step in his management is:

  A. Urgent cardiac catheterization  B. Electrophysiology study

C. Pacemaker  D. Observation  E. Start metoprolol

Explanation:

This patient has classic EKG of third degree heart block. In third degree AV block, no atrial impulses will travel to ventricles. So, atria and ventricles beat independently and have their respective rates; in this case the atrial rate is 80/min and the ventricular rate is around 30/min. These patients are at very high risk for sudden cardiac death and they should be admitted in ICU and permanent pacemaker should be placed as soon as possible. Atropine should always be made available at the bedside. (Option C).

Option A: Any patient who suffers an MI may undergo a cardiac catheterization to evaluate coronary artery disease. This patient has no ‘ST’ elevation; so, in this patient it is not urgent and can be done on an elective basis. The patient's

Page 6: T 20 Cardiology

arrhythmia has to be addressed first as it is a more dangerous condition.

Option B: An EP study does not need to be done in all patients. This patient has a coronary artery disease and suffered a conduction block. The EP study does not change the treatment or the prognosis. EP studies are usually done to identify the arrhythmia and look for the location of conduction block.

Option D: Observation alone is not indicated for any patient with complete heart block.

Option E: Starting beta-blockers is the one thing that you can do to kill this patient immediately; so, obviously it is not the treatment that you would want. Beta-blockers are contraindicated in all types of heart block.

Educational objective:Complete heart block is a dangerous condition, which can cause sudden cardiac death. It requires immediate placement of permanent pacemaker.

52% of people answered this question correctly

Test #20   Q.Id = 976 Question 6 Of 26Item Review

Division : Cardiology

34-year-old previously healthy male, who has recently immigrated from South America (Brazil), presented with a 2 months history of exertional shortness of breath. He denies any chest pain, palpitations, dizziness, or syncope. His only other medical problem was megacolon, which was treated 2 years ago. On examination, there are bilateral basilar crepitations heard and a 1+ pedal edema and mild elevation of jugular venous distention. CBC revealed hemoglobin of 13.9. EKG showed nonspecific repolarization changes and intraventricular conduction abnormalities. Chest x-ray revealed cardiomegaly and pulmonary congestion. Echocardiogram revealed an ejection fraction of 40%. Based on these findings, which of the following is the most likely cause of his problem?

  A. Diphtheric myocarditis  B. Premature coronary artery disease

C. Trypanosoma cruzi  D. Giant cell myocarditis.  E. Rickettsial myocarditis.

Explanation:

Chaga’s disease is a condition caused by insect-borne protozoan called Trypanosoma cruzi, which is a common form of myocarditis in Central and South America. Major clinical manifestations may appear after a latent period of more than a decade. At this stage, patients usually present with

Page 7: T 20 Cardiology

cardiomyopathy, conduction abnormalities, and certain death. This patient has classic features of cardiomyopathy. Almost all these patients will have a have a history megacolon or mega-esophagus.

Educational Objective:Any patient who comes from South America and have findings suggestive of cardiomyopathy should make you think about Chaga’s disease.

70% of people answered this question correctly

Test #20   Q.Id = 1009 Question 7 Of 26Item Review

Division : Cardiology

A 65-year-old white male who is known to have benign prostatic hyperplasia presents to the emergency room with altered mental status. He is febrile and had two episodes of vomiting this morning. He is also known to have type II DM for the last seven years. His medications include aspirin, finasteride, insulin and captopril. On examination, his vitals are as follows: BP: 70/50 mmHg, Temperature: 40C (103F), RR 15/min, pulse rate 110/min. Significant findings on examination are muffled heart sounds and bilateral crackles up to the scapulae. There is no jugular venous distension or ankle edema. He is given IV antibiotics and oxygen. An indwelling urinary catheter is passed and urine output is measured at 20 ml/hour. Urine and blood are sent for culture. Right heart catheterization shows pulmonary capillary wedge pressure of 30 mm Hg and a mean right atrial pressure of 11 mmHg. Based on these findings, what is the most likely cause of shock in this patient?

  A. Right ventricular infarction  B. Pericardial tamponade  C. Constrictive pericarditis

D. Left ventricular dysfunction  E. Septic shock due to urinary tract infection  F. Hypovolemia secondary to vomiting

Explanation:

The above patient is most likely suffering from shock secondary to left ventricular dysfunction. Shock has multiple causes and may result from hypovolemia, sepsis and cardiac causes. The above patient’s shock may be due to any of the above-mentioned etiologies and therefore right heart catheterization is essential to make the diagnosis in this case.

The physical exam findings and the elevated pulmonary capillary wedge pressure are compatible with left ventricular dysfunction.

In the case of right ventricular infarction, pulmonary capillary wedge pressure

Page 8: T 20 Cardiology

would be low and chest examination and chest x-ray would show absence of fluid in the lungs.

In both constrictive pericarditis and pericardial tamponade, right and left sided diastolic pressures are equal and elevated. In the above patient, his pulmonary capillary wedge pressure is elevated but not equal to right atrial pressure thus excluding the diagnosis of pericardial tamponade and constrictive pericarditis.

In cases of septic shock both right atrial pressures and pulmonary capillary wedge pressure are low.

In cases of hypovolemic shock both right atrial pressures and pulmonary capillary wedge pressure are low.

Educational Objective:Recognize the clinical features of cardiogenic shock. Elevated PCWP is the most important finding to concentrate.

15% of people answered this question correctly

Test #20   Q.Id = 630 Question 8 Of 26Item Review

Division : Cardiology  

A 56-year-old white male complains of recurrent chest pain while he is in CCU three days after a large transmural anterior wall myocardial infarction. He received thrombolytic therapy and anticoagulation with heparin. The chest pain is central, 6/10 in severity, radiates to the back, is worsened by deep breathing, and relieved by sitting up. Sublingual nitroglycerin has no effect on his pain. His vitals are, PR: 80/min; BP: 125/70 mmHg; Temperature: 37C(98.6F); RR: 14/min. His lungs are clear to auscultation. Heart sounds are normal with no murmurs and a pericardial friction rub is present. Chest x-ray shows normal sized heart and clear lung fields. EKG shows diffuse new ST segment elevation that is concave upwards. Troponins are still elevated and CK-MB is normal. Based on these findings, what is the most likely diagnosis in this patient?

A. Acute pericarditis  B. Recurrent ischemia  C. Recurrent infarction  D. Dressler’s syndrome  E. Pulmonary embolism

Explanation:

This patient is most likely suffering from acute pericarditis. Typical findings of acute pericarditis in this patient include chest pain, worsened by breathing and improved by leaning forward, presence of a pericardial friction rub and diffuse

Page 9: T 20 Cardiology

ST segment elevation that is concave upwards.

Recurrent ischemia is unlikely as the cause of his chest pain. Ischemic chest pain is not aggravated by breathing and is not relieved by sitting forward.

Recurrent infarction is unlikely as a cause for his chest pain. Pain in cases of infarction would be of the ischemic type and CK-MB would also be elevated. Troponins remain elevated for 1-2 weeks after an acute MI, while CK-MB returns to normal within 48-72 hours. ST elevation will be convex upwards and is seen in certain leads, indicating a certain territory, for example either inferior, anterior or the lateral leads etc.

Dressler’s syndrome, an autoimmune pericarditis, is a late complication of acute MI that usually develops between the second and tenth weeks post MI.

Pulmonary embolism is unlikely in this patient. Patients with pulmonary embolism usually present with sudden onset of dyspnea, chest pain, tachycardia, and tachypnea. This patient received thrombolytic therapy and anticoagulation with heparin and is very unlikely to have a pulmonary embolism.

Educational Objective: Recognize and differentiate acute pericarditis from other causes of recurrent chest pain in the setting of an acute MI.

41% of people answered this question correctly

Test #20   Q.Id = 1003 Question 9 Of 26Item Review

Division : Cardiology

A 32-year-old white female presents with dyspnea, edema and malaise. She complains of breathlessness at rest. She also had episodes of paroxysmal nocturnal dyspnea. She noticed bilateral ankle swelling a week ago that became progressively worse. She is a non-smoker and non-alcoholic. Her past medical history is not significant. She is not taking any medications. Her pulse is 90/min, BP is 140/66 mmHg, and RR is 14/min. Examination shows jugular venous distension, rapidly rising carotid pulse with a sudden collapse and bilateral ankle edema. Her apical impulse is heaving and displaced inferolaterally. Auscultation of lungs shows bilateral crepitations. Cardiac auscultation shows an S3 gallop as well as a high-pitched, blowing, early diastolic decrescendo murmur heard best in the left third intercostal space. Leaning forward and holding the breath in expiration intensifies the murmur. Based on these findings, what is the most likely diagnosis in this patient?

A. Aortic regurgitation  B. Pulmonary regurgitation

Page 10: T 20 Cardiology

  C. Mitral stenosis  D. Tricuspid stenosis  E. Aortic stenosis  F. Tricuspid regurgiation

Explanation:

The most likely diagnosis in this patient is aortic regurgitation. This patient has features of biventricular congestive heart failure secondary to valvular disease. Presence of collapsing (water-hammer) pulse is a clue for aortic regurgitation and occurs due to the hyperdynamic circulation and early diastolic run-off. She has a characteristic murmur of aortic regurgitation that is early diastolic, decrescendo, high-pitched, blowing best heard in the left third intercostal space. The murmur is intensified by leaning forward and holding the breath in expiration (Valsalva).

The murmur of pulmonary regurgitation is early diastolic, decrescendo, high-pitched, blowing, best heard along the left sternal border and it usually develops secondary to pulmonary hypertension. It becomes more prominent with inspiration. Also, the pulmonary component of the second heart sound is loud.

The murmur of mitral stenosis is a mid-diastolic rumble and it is best heard at the apex.

The murmur of tricuspid stenosis is a mid-diastolic rumble and is best heard along the left lower sternal border.

The murmur of the aortic stenosis is ejection systolic and best heard at right 2nd intercostal space.

The murmur of tricuspid regurgiation is pansystolic.

Educational Objective: Recognize the clinical features of aortic regurgitation.

45% of people answered this question correctly

Test #20   Q.Id = 600 Question 10 Of 26Item Review

Division : Cardiology  

A 35-year-old woman who has recently immigrated from Asia presents to the emergency room with acute onset of dyspnea. She denies any cough, chest pain, or fever. She has a history of rheumatic heart disease as a teen. On examination, she has an irregular PR: 97/min; BP: 125/75 mm of Hg; Temperature: 37.2C(98.9F). Examination of precordium shows tapping apical impulse. First heart sound is loud and a mid-diastolic rumble is heard at the apex. Crepitations are present in both lung fields. EKG shows an irregularly,

Page 11: T 20 Cardiology

irregular heart rhythm and the absence of ‘P’ waves. Which of the following is most likely the cause of abnormal heart rhythm in this patient?

A. Left atrial dilatation  B. Right atrial dilatation  C. Left ventricular dilatation  D. Left ventricular hypertrophy  E. Pulmonary hypertension

Explanation:

This patient is in congestive heart failure due to atrial fibrillation (the irregularly, irregular heart rate and absence of ‘P’ waves) which itself is secondary to mitral stenosis (H/O rheumatic fever, mid diastolic rumble, and loud S1). Left atrial enlargement that results from mitral stenosis predisposes the patient to the development of atrial fibrillation. Left atrial enlargement causes atrial fibrillation by increasing the refractory period as well as the action potential duration and it is responsible for all cases of atrial fibrillation occurring in the setting of MS.

Left ventricular dilatation and left ventricular hypertrophy do not occur in isolated MS.

Right atrial dilation is a late finding in the course of MS and it occurs when right sided heart failure takes place secondary to pulmonary hypertension. Right atrial dilatation may cause atrial fibrillation also, but in cases of MS, the cause for atrial fibrillation is usually left atrial dilation that occurs earlier in the course of disease. Pulmonary HTN itself does not cause atrial fibrillation, but the right atrial enlargement that results from it, does.

Educational Objective: Atrial fibrillation in MS is due to left atrial dilatation.

56% of people answered this question correctly

Test #20   Q.Id = 592 Question 11 Of 26Item Review

Division : Cardiology  

A 56-year-old white male presents with dyspnea for the last 3 months. His dyspnea was initially exertional but it has worsened progressively and now he is breathless even at rest. He denies any chest pain or ankle swelling. He has been smoking one-pack/day cigarettes for the last 30 years and has been drinking alcohol heavily for the last 10 years. He is not taking any medication. His mother died of breast cancer at 57. His vitals are, PR: 86/min, BP: 113/76mm of Hg; Temperature: 37.1C(98.9F); RR: 13/min. On auscultation of his precordium an S3 is heard, but there are no murmurs. Chest auscultation reveals bilateral basal crepitations. Chest x-ray shows marked cardiac silhouette enlargement and pulmonary venous congestion. EKG shows non-specific ST-T

Page 12: T 20 Cardiology

wave changes. Echocardiography shows a dilated left ventricle and systolic dysfunction (EF of 25-30%). CBC shows hematocrit of 32%, WBC count of 6,000/microL, and platelet count of 60,000/microL. Peripheral blood smear shows MCV of 101 fL. LFTs show AST of 180U/L and ALT of 66 U/L. The findings of cardiac catheterization and coronary angiography are not compatible with the diagnosis of ischemic cardiomyopathy. Which of the following measures is most likely to reverse his heart failure?

  A. Cessation of cigarette smokingB. Abstinence from alcohol

  C. Reduced salt intake  D. Use of ACE inhibitors  E. Use of digoxin

Explanation:

This patient is most likely suffering from dilated cardiomyopathy secondary to alcoholism. Findings of thrombocytopenia, macrocytosis, and elevated transaminases are all suggestive of alcoholism in this patient. His cardiomyopathy could be due to ischemia but coronary angiography excluded this diagnosis in this patient. Total abstinence from alcohol is the mainstay of alcoholic cardiomyopathy management and it may reverse this condition if it is employed earlier in the course of the disease.

Reduced salt intake only improves symptoms of heart failure and does not reverse the disease process itself.

Digitalis is most useful in those heart failure patients who have systolic dysfunction and have rapid ventricular rates due to atrial flutter or atrial fibrillation. It proves to be useful in these cases by its positive inotropic effect and negative dromotropic (slowing AV conduction). Use of digitalis has not been shown to provide any survival advantage in patients with congestive heart failure and has not been shown to reverse the disease process.

ACE inhibitors have been shown to slow the progression of heart failure but they don’t reverse the disease process of alcoholic cardiomyopathy. ACE inhibitors should be given to all patients with heart failure who have systolic dysfunction unless they are contraindicated or the patients can’t tolerate them.

Cigarette smoking is a risk factor for coronary heart disease and its cessation should be encouraged in all patients with heart failure. Cessation of cigarette smoking does not reverse the disease process of alcoholic cardiomyopathy.

Educational Objective: Know how to diagnose and manage a patient with congestive heart failure due to alcoholic dilated cardiomyopathy.

Page 13: T 20 Cardiology

34% of people answered this question correctly

Test #20   Q.Id = 606 Question 12 Of 26Item Review

Division : Cardiology  

A 40-year old female immigrant from Asia comes for evaluation of her dyspnea. Her dyspnea was mild 6 months ago but it has now progressed to dyspnea, even at rest. She denies any chest pain, syncope, or palpitations. She is a non-smoker and non-alcoholic. Her past medical history is significant for pulmonary tuberculosis. Her PR: 82/min; BP: 135/70 mmHg; Temperature: 37.1C(98.8F); RR: 14/min. On her examination, significant physical findings are jugular venous distension, bilateral ankle edema, and tender hepatomegaly. Chest x-ray shows pericardial calcifications. Which of the following set of physical findings is most likely to be present in this patient?

A. Early third heart sound and inspiratory increase in jugular venous pulse  B. Water hammer pulse and pistol shot femorals  C. Tapping apex beat and malar flush  D. Pulsus paradoxus and hypotension  E. Pansystolic murmur at left lower sternal border

Explanation:

This patient, most likely, has constrictive pericarditis. The finding of pericardial calcifications on the chest X-ray is an important clue. The etiology of pericarditis is probably from her prior history of Tuberculosis. The early third heart sound, that is also called pericardial knock and the inspiratory increase in the jugular venous pressure (Kussmaul’s sign), are important physical findings of constrictive pericarditis. Kussmaul’s sign is also present in right sided heart failure, severe tricuspid regurgitation, right ventricular infarction and cardiac tamponade.

Water hammer or collapsing pulse and pistol shot femoral pulses are diagnostic clues to aortic regurgitation. These physical findings occur due to a hyperdynamic circulation and early diastolic runoff of aortic insufficiency.

Tapping apex beat and malar flush are important physical findings of mitral stenosis. Pulsus paradoxus is defined as greater than 10-mmHg fall of the systolic blood pressure during inspiration.

Pulsus paradoxus and hypotension point toward the diagnosis of pericardial tamponade. The former is uncommon in constrictive pericarditis unless an effusion is present. It may also be present in severe airway obstruction and superior vena cava obstruction.

A pansystolic murmur at the left sternal border is usually seen in tricuspid regurgitation.

Page 14: T 20 Cardiology

Educational Objective: Know the characteristic clinical findings of constrictive pericarditis.

35% of people answered this question correctly

Test #20   Q.Id = 1746 Question 13 Of 26Item Review

Division : Cardiology

patient is referred to the urologist for urethral stricture dilatation which may result in bacteremia and cause infective endocarditis in predisposed patients. Which of the following patients would not require antibiotic prophylaxis to prevent infective endocarditis?

A. A 50-year-old male with pacemaker  B. A 17-year-old male with cyanotic heart disease  C. A 50-year- old female with previous infective endocarditis  D. A 40-year-old male with mitral valve prolapse with regurgitation  E. A 30-year-old male with prosthetic aortic valve

Explanation:

The need of antibiotic prophylaxis for IE depends on the cardiac abnormality and the procedure causing bacteremia. Certain cardiac conditions are more susceptible to endocarditis than others. The risk of bacteremia and the likely organisms vary according to the procedure being performed. Therefore, the decision whether or not to prophylax, and the choice of antibiotics, depends both on the cardiac abnormality and on the procedure.

Cardiac abnormalities are high risk, moderate risk or negligible risk. Antibiotic regimen is different for GI and GU procedures in high-risk and moderate-risk patients. Antibiotic prophylaxis is not recommended in moderate-risk patients with some low-risk procedures. Generally, we can say that prophylaxis is recommended for high-risk conditions and is optional for moderate-risk conditions.

Conditions, which put a patient at high risk to develop infective endocarditis include:

1. All prosthetic heart valves.   2. Any history of previous bacterial endocarditis.  3. Complex cyanotic congenital heart disease and surgically- constructed

systemic pulmonary shunts. 

Conditions, which put a patient at moderate risk of developing infective

Page 15: T 20 Cardiology

endocarditis include:

1. Congenital cardiac malformations not falling into the high or negligible risk categories (such as PDA, VSD, ostium primum ASD, bicuspid aortic valve and coarctation). 

2. Acquired valvular heart disease (such as rheumatic heart disease, valvular stenosis and regurgitation). 

3. MVP with regurgitation and/or myxomatous leaflets.  4. Hypertrophic cardiomyopathy. 

Conditions that do not require IE prophylaxis include:

1. Isolated ostium secundum ASD and surgically-repaired ASD, VSD and PDA (beyond six months and without sequelae). 

2. Mitral valve prolapse without mitral regurgitation and without thickened leaflets. 

3. Innocent or physiologic murmurs (echo required in the adult population to rule out valvular lesion). 

4. Cardiac pacemakers and defibrillators.  5. History of isolated bypass surgery, history of Kawasaki disease without

valvular dysfunction and history of rheumatic fever without valvular dysfunction.

Educational Objective:Patients with artificial pacemakers and defibrillators do not require prophylaxis for infective endocarditis.

35% of people answered this question correctly

Test #20   Q.Id = 634 Question 14 Of 26Item Review

Division : Cardiology

A 76-year-old male comes to the emergency room with sudden onset of severe chest pain. The pain is 10/10 in severity, tearing in quality, and radiates to the back. He is a known diabetic for 10 years and is hypertensive for 8 years. His medications include insulin, metoprolol and hydrochlorothiazide. His vitals are, PR: 86/min; BP: 166/96 mmHg in his right arm and 136/70 in his left arm; Temperature: 37C(98.6F); RR: 14/min. On auscultation, lung fields are clear and heart sounds are normal without any murmurs. Chest x-ray and EKG are normal. Bases on these findings, which of the following is the most likely diagnosis of this patient?

  A. Angina pectoris  B. Myocardial infarction  C. Acute pericarditis

Page 16: T 20 Cardiology

D. Aortic dissection  E. Pulmonary embolism

Explanation:

This patient is most likely suffering from aortic dissection. The most frequent predisposing condition for aortic dissection is hypertension. Sudden onset of severe tearing pain, with radiation to the back, is a typical presentation of aortic dissection. The finding of a difference of more than 30 mmHg in the blood pressure readings between two arms is another important clue for the diagnosis of aortic dissection.

Pain of IHD usually feels like pressure and it typically radiates to the jaw, left shoulder or left arm. Pain of angina occurs with exertion and is relieved with rest and sublingual nitroglycerin. Pain of MI occurs at rest and is not fully relieved by sublingual nitroglycerin. EKG usually provides evidence for ischemic heart disease with T-wave inversion in angina pectoris, and ST segment elevation or ST segment depression in cases of MI. The nature of pain in the above patient with absent EKG changes makes the diagnosis of angina, or MI, very unlikely.

Pain in acute pericarditis is pleuritic, is worsened by deep breathing and coughing and is alleviated by leaning forward. EKG may show ST segment elevation that is concave upwards without any reciprocal changes.

Pulmonary embolism usually presents with sudden onset of dyspnea, chest pain, tachycardia, and tachypnea. There is usually a history of risk factors like immobilization, surgery of hip or femur etc. that predisposes the development of pulmonary embolism.

Educational Objective:Suspect aortic dissection as a cause of tearing chest pain in the setting of HTN and BP difference in the 2 arms.

69% of people answered this question correctly

Test #20   Q.Id = 1634 Question 15 Of 26Item Review

Division : Cardiology

A 72-year-old female is admitted to the ICU with severe chest pain. The initial set of cardiac enzymes are positive and her ECG reveals an anterior wall MI. She is being treated with aspirin, t-PA, metoprolol, nitroglycerine drip, and morphine. Two hours later, her telemetry monitor rhythm changes and is shown below. The next step in her management is:

Page 17: T 20 Cardiology

  A. IV lidocaineB. Defibrillation

  C. IV digoxin  D. IV amiodarone  E. Stat echocardiogram  F. STAT check of potassium and magnesium

Explanation:

Ventricular fibrillation can occur as a consequence of or in association with a serious illness, cardiovascular disease or non-cardiovascular disorder. Ventricular fibrillation reflects a total disorganization of ventricular electrical activity and is recognized on the ECG by fibrillatory waves and absence of regular QRS waves. It should be differentiated from cardiac asystole, which is characterized by a flat base line. All patients have no BP at the onset of ventricular fibrillation and the most immediate step is to defibrillate as soon as possible. The energy required may be 200-360 joules. More than a few minutes of delay can lead to permanent brain injury and death (Option B).

(Option A): Lidocaine is an excellent agent for ventricular arrhythmias. In the presence of ventricular fibrillation, cardioversion is the first modality of therapy. If cardioversion does not initially work, patients may require a lidocaine bolus followed by a second shock.

(Option C): Digoxin has no role in ventricular fibrillation.

(Option D): Amiodarone is an excellent drug for stable ventricular tachycardia and maintaining normal sinus rhythm following a ventricular arrhythmia. However, when ventricular fibrillation occurs, the first choice is cardioversion. Amiodarone can be loaded to maintain the normal sinus rhythm. Amiodarone is being used more frequently (drug of choice) over lidocaine for ventricular arrhythmias.

(Option E): Echo may be done later to assess the heart function but it does not change the management of ventricular fibrillation. Ventricular arrhythmias are more prone in patients with ischemic heart disease and in those with low ejection fractions.

(Option F): When ventricular fibrillation occurs, the first thing is to defibrillate the patient. Electrolytes and blood gas analysis can be done during the procedure but should not delay shocking. Ventricular arrhythmias may occur in the presence of hypokalemia.

Educational objective:The treatment of ventricular fibrillation is STAT defibrillation with 200-360 joules. If defibrillation fails, lidocaine or amiodarone (drug of choice) can be loaded and the patient shocked again. Epinephrine can sensitize the heart and lower the

Page 18: T 20 Cardiology

threshold for conversion.

35% of people answered this question correctly

Test #20   Q.Id = 1856 Question 16 Of 26Item Review

Division : Cardiology

50-year-old woman presents to your office complaining of lower extremity edema that started several weeks ago, and slowly progressed thereafter. Her past medical history is significant for hypertension, treated with metoprolol for 2 years.  Amlodipine was added recently because of inadequate control of BP with metoprolol alone. She does not smoke or consume alcohol. She has no known drug allergies. Her blood pressure is 130/80mm Hg and her heart rate is 64/min. The physical examination reveal bilateral symmetric 3+ pitting edema of both lower extremities, without any skin changes or varicosities. Her neck vein pulsation is normal. Other physical findings are within normal limits. Her laboratory studies reveal the following:

Serum albumin                     4.5 g/dLTotal serum bilirubin             0.8 mg/dLSerum sodium                      140 mEq/LSerum potassium                  4.0 mEq/LSerum creatinine                   0.8 mg/dL

Urinalysis is within normal limits. What is the most likely cause of the edema in this patient?

  A. Heart failure  B. Liver disease  C. Renal disease  D. Venous insufficiency

E. Side effect of her medicationsExplanation:

It is important to remember that peripheral edema is a common side effect of the treatment with dihydropyridine Ca-channel antagonists like amlodipine. These medications can cause significant edema, as in this patient, due to the property to dilate peripheral blood vessels; it is NOT an allergic reaction! If the edema is significant, the drug should be discontinued. The clues to the correct diagnosis in this case are the therapy with a dihydropyridine Ca-channel antagonist, amlodipine initiated recently, and normal physical examination with normal laboratory findings. Of course, it is necessary to exclude other more serious causes of edema.

Congestive heart failure (Choice A) is an important cause of lower extremity

Page 19: T 20 Cardiology

edema, but the absence of other symptoms of heart failure (e.g., dyspnea, orthopnea) and physical findings, especially elevated neck vein pulsation and liver enlargement, makes this diagnosis unlikely.

Liver diseases (Choice B) can cause lower extremity edema, but usually ascites dominates over peripheral edema, and abnormal laboratory findings characteristic of liver dysfunction are present (e.g., hypoalbuminemia, hyperbilirubinemia).

Renal diseases (Choice C) can cause peripheral edema due to massive proteinuria (as with nephrotic syndrome) or fluid retention (such as during acute nephritic syndrome). The absence of proteinuria, hypoalbuminemia, and normal creatinine level are inconsistent with renal causes of the edema.

Venous insufficiency (Choice D) is a possibility in this case.  Symmetric edema without skin changes and varicosities, however, argues against venous insufficiency.

Educational objective:Dihydropyridine Ca-channel antagonists can cause peripheral edema and should always be considered in the differential diagnosis of this condition, along with other causes, such as heart failure, renal disease and venous insufficiency.

36% of people answered this question correctly

Test #20   Q.Id = 612 Question 17 Of 26Item Review

Division : Cardiology

A 15-year-old white male, who has recently immigrated from Asia, is found to have a heart murmur during his routine physical examination. He has a documented history of acute rheumatic fever, without any evidence of carditis, a year ago. He is otherwise completely asymptomatic. His vitals are stable. His apex beat is tapping. Cardiac auscultation shows a loud first heart sound, opening snap and mid-diastolic rumble at the apex. His lungs are clear to auscultation. Chest x-ray and EKG are unremarkable. What is the next most appropriate step in the management of this patient?

A. Penicillin prophylaxis  B. Refer to cardiology for balloon valvotomy  C. Refer to cardiothoracic surgeon for mitral valve replacement  D. Consider anticoagulation

Explanation:

This patient is suffering from rheumatic heart disease with pure mitral stenosis. Although he is asymptomatic at this stage, his disease is likely to be progressive. It has been found that prevention of recurrent attacks of rheumatic

Page 20: T 20 Cardiology

fever may slow down the progression of mitral stenosis. Therefore, penicillin prophylaxis with monthly IM injection of benzathine penicillin is recommended in adolescent patients in whom the recurrence risk is high. Patients with, or without, carditis need penicillin prophylaxis, although the recommended duration of prophylaxis is 10 years in the former and 5 years in the latter. Patients with mitral stenosis are also at high risk for thromboembolism. Anticoagulation is indicated if they have a history of an embolic event or they have intermittent or chronic atrial fibrillation.

Surgical intervention for mitral stenosis is indicated when these patients develop symptoms of NYHA functional class III or IV. Balloon valvotomy is the procedure of choice when there is favorable valve morphology.

Mitral valve replacement is considered to be the last resort for the treatment of MS because of the high perioperative mortality and morbidity.

Educational Objective: Know that prevention of recurrent attacks of rheumatic fever with antibiotic prophylaxis may slow down the progression of mitral stenosis in adolescents. Asymptomatic MS do not require any treatment except penicillin prophylaxis.

46% of people answered this question correctly

Test #20   Q.Id = 17 Question 18 Of 26Item Review

Division : Cardiology  

A 26-year-old primigravida at 20th week gestation presents to the emergency room with a sudden onset of tearing chest pain radiating to her back and left arm. The patient is pale and diaphoretic. Her PR: 116/min; BP: 190/100 in left arm, and 80/60 in right arm; RR: 36/min. Her cardiac examination reveals a diastolic murmur along the left sternal border. Her previous prenatal care is not known. She is a smoker with a 10 pack/year history and drinks alcohol. Her ECG reveals mild left axis deviation and ST segment depression in lead II, III, and AVF. What is the most appropriate next step in the management of this patient?

  A. Obtain CK-MB and Troponin levels  B. Transesophageal echocardiogram (TEE)  C. Transthoracic echocardiogram (TTE)

D. Antihypertensive treatment  E. CT scan of chest  F. Chest - X ray

Explanation:

Patient is most likely suffering from dissection of the aorta. Pregnancy is associated with dissection of aorta, probably due to changes in connective

Page 21: T 20 Cardiology

tissue associated with pregnancy. Other risk factors for dissection include a bicuspid aortic valve, coarctation of aorta, and Marfan’s syndrome. Aortic dissection is a medical emergency and should be diagnosed and treated. If hypertension is present it should be treated aggressively before any diagnostic studies are done. ECG changes are due to involvement of the coronary ostia (usually the right coronary artery), which lead to inferior wall ischemia, manifested by the abnormalities on ECG.

Choice (A): Obtaining cardiac enzymes is incorrect because the patient doesn’t have any risk factors for an MI. Her ECG changes are secondary to ischemia and not due to infarction.

Choice (B): TEE is the initial investigation of choice to diagnose suspected aortic dissection. But it is indicated only after hypertension treatment is initiated.

Choice (C): Transthoracic echocardiogram is not used to diagnose aortic dissection.

Choice (E): CT scan of chest also has high sensitivity and specificity and can be used instead of the transesophageal echo.

Choice (F): Chest - X ray also reveals aortic dissection as mediastinal enlargement but still not the 1st step in this patient.

Educational Objective:Antihypertensive management should be the first step in patients with aortic dissection with hypertension.

24% of people answered this question correctly

Test #20   Q.Id = 589 Question 19 Of 26Item Review

Division : Cardiology  

A 20-year-old white male comes with the complaint of exertional dyspnea for the last 2 months. He denies any chest pain, cough, fever or syncope. He is not taking any medicines. He does not smoke and drinks alcohol only on weekends. One of his brothers died of a heart disease when he was 22. His vitals are stable and he is afebrile. On examination, there is no edema, jugular venous distension or hepatomegaly. His lungs are clear to auscultation. On auscultation of the precordium, a harsh crescendo-decrescendo grade III systolic murmur is heard most prominently along the left lower sternal border. The murmur is intensified by Valsalva maneuver and attenuated by leg elevation. What should you order next in this patient?

  A. Electrocardiogram

Page 22: T 20 Cardiology

B. Echocardiography  C. Cardiac catheterization  D. Stress testing  E. MRI  F. Exercise thallium SPECT imaging

Explanation:

This patient is most likely suffering from hypertrophic cardiomyopathy (HCM). HCM can be asymptomatic or it may present with symptoms of exertional dyspnea, chest pain or syncope. The typical auscultatory finding is a harsh diamond shaped systolic murmur best heard at the left lower sternal border. The murmur of HCM increases in intensity with the Valsalva maneuver and is attenuated by leg elevation. The best way to diagnose HCM is echocardiography, which usually shows asymmetrical interventricular septal hypertrophy. In most cases, the thickness of the septum is 1.3 times greater the thickness of left ventricular free wall. Systolic anterior motion of the mitral valve is noted if an outflow pressure gradient is present.

Cardiac catheterization is not required for the diagnosis of HCM but it helps to quantify the pressure gradient and to exclude coexistent coronary artery disease as a cause of chest pain in older patients.

ECG shows evidence of left ventricular hypertrophy but it is insufficient to make a diagnosis of HCM. Other abnormal ECG findings include prominent Q waves in inferior and lateral leads due to septal depolarization of the hypertrophied myocardium.

Exercise testing is not diagnostic of HCM but it provides a measure of patient’s physical limitations.

In some cases, echocardiography fails to establish wall thickness and extent of hypertrophy. MRI is an alternative diagnostic study for HCM when echocardiography is suboptimal.

Exercise thallium scintigraphy is useful for detecting ischemic but viable myocardium.

Educational Objective: The investigation of choice for the diagnosis of HCM is echocardiography.

66% of people answered this question correctly

Test #20   Q.Id = 632 Question 20 Of 26Item Review

Division : Cardiology  

A 63-year-old white male presents to the ER with sudden onset of severe retrosternal chest pain associated with nausea, vomiting and diaphoresis. His

Page 23: T 20 Cardiology

EKG shows ST segment elevation in leads aVL and I. The patient was treated with thrombolytic therapy in the ED. He is known to have insulin requiring diabetes mellitus for the past 3 years as well as hypertension for the past 2 years. His medications include insulin and enalapril. Which of the following set of medications has been shown to reduce mortality after an acute MI?

  A. Nitrates, beta blockers, and aspirin  B. ACE inhibitors, nitrates, and beta blockers  C. Magnesium, beta blockers, and aspirin  D. Oxygen, morphine, aspirin, and nitrates

E. Aspirin, beta blockers, and ACE inhibitorsExplanation:

Of all the above choices, only ACE inhibitors, aspirin, and beta-blockers have been shown to reduce mortality after acute MI.

The proposed mechanism for ACE inhibitors is the reduction of ventricular remodeling and thus reduced incidence of left ventricular dysfunction / aneurysmal dilatation after MI. They are recommended in all patients of acute MI with CHF as well as to those who are hemodynamically stable and have ST segment elevation or left bundle branch block. They are usually started within the first 24 hours of acute MI and are continued indefinitely in those who have CHF, HTN or LV dysfunction.

Aspirin has also been shown to be effective and should be given in all patients, as early as possible in the setting of an acute MI, because it has been shown to reduce mortality by as much as 30%.

3 doses of IV beta-blockers, followed by oral beta-blockers in doses titrated for a heart rate of 50-60, have also been shown to reduce the mortality significantly in acute coronary syndromes.

Morphine is an effective analgesic and is used for pain relief in the setting of an acute MI. Oxygen is useful only when pulse oximetry shows hypoxemia. Its routine use in all patients of acute MI is not cost effective. Nitrates are used only for pain relief in the setting of acute MI and they don’t incur any survival advantage. IV nitrates should be avoided when there is hypotension or evidence of right ventricular infarction. IV magnesium is indicated only when initial testing shows low serum magnesium level. Routine use of IV magnesium in all MI patients does not provide any survival benefit.

Educational Objective:Aspirin, ACE inhibitors, and beta-blockers have been shown to reduce mortality in the setting of acute MI.

51% of people answered this question correctly

Page 24: T 20 Cardiology

Test #20   Q.Id = 594 Question 21 Of 26Item Review

Division : Cardiology

A 60-year-old female presents with dyspnea and ankle edema. Her dyspnea was mild 6 months ago and used to occur with moderate exertion but now has progressed to dyspnea, even at rest. She denies any chest pain, syncope, or palpitations. She does not smoke or drink alcohol. She denies diabetes mellitus, hypertension, or hyperlipidemia. Her vitals are, PR: 80/min; BP: 130/70 mmHg; Temperature: 37.1C(98.9); RR: 14/min. Examination is significant for jugular venous distension, bilateral ankle edema, and tender hepatomegaly. Kussmaul’s sign is positive and pulsus paradoxus is negative. Chest auscultation shows bibasilar crackles. Heart sounds are distant and there is no murmur. Chest x-ray shows mild cardiomegaly and a right-sided pleural effusion. EKG shows low voltage QRS complexes and nonspecific ST-T wave changes. Echocardiography shows symmetrical thickening of left ventricular walls, normal ventricular size, and slightly reduced systolic function. CT scan of the chest shows normal pericardial thickness. Based on these findings, which of the following is the most likely diagnosis in this patient?

  A. Dilated cardiomyopathyB. Restrictive cardiomyopathy

  C. Hypertrophic cardiomyopathy  D. Cardiac tamponade  E. Constrictive pericarditis

Explanation:

Restrictive cardiomyopathy is characterized by severe diastolic dysfunction due to a stiff ventricular wall. Chest x-ray shows only mild enlargement of the cardiac silhouette. Echocardiography usually shows a symmetrically thickened ventricle wall, normal or slightly reduced left ventricle size and normal or near normal systolic function. Kussmaul’s sign may also be present. The apical impulse is easily palpable in restrictive cardiomyopathy as opposed to constrictive pericarditis. Restrictive cardiomyopathy is difficult to differentiate from constrictive pericarditis. With constrictive pericarditis, chest x-rays may show pericardial calcifications and the CT scan usually shows increased thickness of pericardium. Kussmaul’s sign may be positive in both conditions. With constrictive pericarditis, the thickness of myocardium will be normal.

Dilated cardiomyopathy is characterized by impaired systolic function of left and right ventricle leading to progressive cardiac enlargement. Chest x-ray shows marked or moderate enlargement of cardiac silhouette. Echocardiography shows systolic dysfunction and left ventricular dilatation with normal thickness of the ventricular wall.

Hypertrophic cardiomyopathy is characterized by asymmetric left ventricular hypertrophy. In HCM, a harsh systolic murmur best heard at the left sternal

Page 25: T 20 Cardiology

border is also present. Chest x-ray shows mild enlargement of cardiac silhouette. Echocardiography shows vigorous systolic function, asymmetric septal hypertrophy and in some cases systolic anterior motion of the mitral valve. Due to the hypertrophy of the left ventricular wall, there is diastolic dysfunction.

In cardiac tamponade, there is sinus tachycardia and hypotension. Pulsus paradoxus is also present in this condition. There is jugular venous distension with a prominent 'y' descent.

Educational Objective: Know how to diagnose restrictive cardiomyopathy and how to differentiate it from other cardiac conditions that may present similarly.

38% of people answered this question correctly

Test #20   Q.Id = 1713 Question 22 Of 26Item Review

Division : Cardiology  

A 55-year-old male is admitted to the ICU after being involved in a motor vehicle accident. He required an exploratory laparotomy for a bowel perforation. Two days after surgery he remains hypotensive and requires both volume and pressor support. At night, the nurse informs you that the patient’s fingers are blue and cold. The photograph is shown below. Which of the following is the most liklely cause of this finding?

  A. Septic embolism  B. Raynaud’s syndrome

C. Norepinephrine  D. Aortic occlusion  E. Thrombosis of forearm vessels

Explanation:

The above presents a case of a patient who is hypotensive and receives fluid and pressor support. Later, he is found to have all digits blue. The most likely cause of this patient’s pathology is related to use of norepinephrine. Norepinephrine is a powerful vasoconstrictor and can lead to decreased blood supply to both the lower and upper extremity. Norepinephrine is a powerful pressor drug and frequently used to revive patients in shock. In addition to decreasing blood supply to the digits, other organs like the kidneys can also be affected. For this reason, the use of this vasoconstrictor is limited in the ICU. Patients who have prior atherosclerotic and peripheral vascular disease are

Page 26: T 20 Cardiology

most sensitive to norepinephrine (Option C).

Option A: Emboli to all five digits of the hand is very rare. Embolus in the upper extremity is very rare and is occasionally related to procedures done in the brachial artery. Emboli usually affects one or two digits and only the tips of the finger will be blue. Large emboli will usually affect the entire blood supply to the radial or ulnar arteries and the bluishness/cyanosis will be visible at the wrist or forearm. Even a complete occlusion of the radial artery is well tolerated by most patients.

Option B: Raynaud’s syndrome is seen in a few vasculitic conditions and is due to spasm of the vessels. The condition is not selective to one hand only and is worse in cold weather. Most patients have a history of lupus, scleroderma or CREST. Raynaud’s is typically seen in females. Diagnosis is based on clinical findings and treatment is the use of calcium channel blockers.

Option D: Occlusion of the aorta does not produce selective bluishness of the fingers. Sometimes, occlusion of the subclavian artery can occur in thoracic outlet syndrome and leads to a cool hand. In this case, the entire hand is cold and painful, requiring urgent treatment. Occlusion of the aorta generally occurs in the abdominal area and leads to decreased blood flow to the lower extremities. Acute aortic occlusion can compromise blood flow to the lower extremities and abdominal organs.

Option E: Thrombosis of the forearm vessels is very rare and does not present acutely with a cold hand. Thrombosis of the forearm vessels may be seen in Buerger’s disease. Buerger’s disease is seen in young males who are heavy smokers. Digital ischemia and gangrene are common features.

Educational Objective:Bluish discoloration and cool fingers in the ICU are a common finding after use of norepinephrine for hypotension.

12% of people answered this question correctly

Test #20   Q.Id = 1008 Question 23 Of 26Item Review

Division : Cardiology

A 64-year-old white female who is known to have congestive heart failure presents for the evaluation of nausea, vomiting, vague abdominal pain, diarrhea, fatigue and blurred vision for the last one-day. She denies any worsening of her breathlessness, chest pain, fever, productive cough or urinary complaints. She had an anterior MI one year ago and later on developed congestive cardiac failure secondary to left ventricular systolic dysfunction. She is currently on aspirin, digoxin, furosemide, enalapril and metoprolol. On

Page 27: T 20 Cardiology

examination her vitals are B.P: 120/80 mm Hg, R.R: 15/min, PR: 66/min. Physical examination is unremarkable. Chest x-ray shows enlargement of cardiac silhouette and normal lung fields. She is suspected to have digitalis toxicity and her blood is sent for digoxin levels. EKG is ordered. Which of the following EKG finding is most specific for digitalis toxicity?

  A. Atrial flutter  B. Atrial fibrillation  C. Mobitz type II AV block  D. ST segment depression  E. First degree AV block

F. Atrial tachycardia with AV blockExplanation:

Digitalis produces a number of EKG abnormalities and arrhythmias. Some of these abnormalities occur at therapeutic serum levels of digoxin and are called digitalis effect. Other EKG abnormalities occur at toxic serum levels of digoxin. Digitalis is a frequently used drug in the treatment of congestive cardiac failure and signs of its toxicity including EKG findings are very important. Digitalis produces these EKG abnormalities and arrhythmias due to its depressant action on AV node and SA node and by increasing myocardial automaticity. ST segment depression, T wave inversion and first degree AV block can occur at therapeutic levels of digoxin and they do not represent digitalis toxicity and therefore there is no need for discontinuation of the drug.

Atrial flutter, atrial fibrillation and Mobitz type II block are rare arrhythmias induced by digitalis toxicity.

Atrial tachycardia along with variable degree of AV block is the most important EKG finding of digitalis toxicity and it has a high specificity for digitalis toxicity.

Educational Objective Know the electrocardiographic manifestations of digitalis toxicity.

28% of people answered this question correctly

Test #20   Q.Id = 591 Question 24 Of 26Item Review

Division : Cardiology  

40-year-old African American male presents with two episodes of syncope after strenuous exercise. He also complains of breathlessness on exertion. He quit smoking 15 years ago and he does not drink alcohol. He is not taking any medication. He denies any chest pain, cough, or fever. His vitals are stable and he is afebrile. On examination there is no edema, ascites, hepatomegaly or jugular venous distension. His lungs are clear to auscultation. Auscultation of his precordium shows a harsh diamond shaped systolic murmur best heard at the left lower sternal border. A diagnosis of hypertrophic cardiomyopathy is

Page 28: T 20 Cardiology

established based on echocardiogram findings. Which of the following is true about the murmur of hypertrophic cardiomyopathy?

  A. Decrease in intensity by ValsalvaB. Decrease in intensity by handgrip

  C. Decrease in intensity by standing after squatting  D. Increase in intensity by phenylephrine  E. Increase in intensity by leg elevation

Explanation:

In almost 25% of cases of hypertrophic cardiomyopathy there is dynamic obstruction to left ventricular outflow tract caused by apposition of the anterior mitral leaflet to the septum (the systolic anterior motion of the mitral valve on the echocardiogram). As a result of this dynamic obstruction, filling pressures are further elevated and forward output is compromised. This outflow gradient is increased by maneuvers that reduce the cavity size of left ventricle. Valsalva maneuver and standing after squatting are two maneuvers that decrease left ventricle volume thus increasing the gradient and intensifying the associated systolic murmur. On the other hand, handgrip increases the systemic arterial resistance and thus decreases the gradient and associated systolic murmur. Phenylephrine also decreases the murmur by increasing systemic arterial pressure. Leg elevation increases the left ventricular volume and thus decreases the gradient and the associated murmur.

Educational Objective: Know the physiology of HCM and be able to correctly identify the factors that increase or decrease the intensity of the murmur in HCM.

38% of people answered this question correctly

Test #20   Q.Id = 1997 Question 25 Of 26Item Review

Division : Cardiology  

32-year-old Caucasian female comes to the physician because of a one-week history of fatigue, progressive worsening of shortness of breath, and swelling of feet. She denies any chest pain. She has no other medical problems except a recent cold two weeks ago. She is not taking any medications. Her temperature is 36.7 C (98 F), blood pressure is 110/65 mmHg, pulse is 90/min, and respirations are 20/min. Bilateral basal crackles, elevated jugular venous pressure, and 2+ bilateral pitting edema of the ankles are noted. Complete blood count is unremarkable. Transthoracic echocardiogram of her heart will most likely show?

  A. Concentric hypertrophy of the heart  B. Eccentric hypertrophy of the heart  C. Mitral stenosis

Page 29: T 20 Cardiology

  D. Hypokinesia of the inferior wall  E. Vegetations on aortic valve  F. Asymmetric septal hypertrophy

G. Dilated ventricles with diffuse hypokinesia  H. Primary pulmonary hypertension

Explanation:

The clinical presentation of this patient is suggestive of congestive heart failure (CHF). A history of recent upper respiratory tract infection followed by sudden onset of cardiac failure in an otherwise healthy patient is suggestive of dilated cardiomyopathy, most likely secondary to acute viral myocarditis.

Dilated cardiomyopathy is the end result of myocardial damage produced by a variety of toxic, metabolic, or infectious agents. Viral or idiopathic myocarditis is most commonly seen following Coxsackie B infection. Viral myocarditis is seen in about 3.5-5 % of patients infected with Coxsackie B virus. Other viruses implicated are adenovirus, cytomegalovirus, echovirus, hepatitis C, influenza virus, parvovirus B-19, and Epstein-Barr virus. The diagnosis is made by echocardiogram, which typically shows dilated ventricles with diffuse hypokinesia resulting in low ejection fraction (systolic dysfunction). Viral myocarditis can cause dilated cardiomyopathy by direct viral damage, as well as sequel of humoral or cellular immune responses to persistent viral infection.

(Choice A) Concentric hypertrophy of the heart is seen following chronic pressure overload, as in valvular aortic stenosis or untreated hypertension; however, these are chronic conditions and would not have acute onset of heart failure.

(Choice B) Eccentric hypertrophy of the heart is seen following chronic volume overload, as seen in valvular regurgitation; however, this is a chronic condition and would not have acute onset of heart failure.

(Choice C) Mitral stenosis is characterized by middiastolic murmur and opening snap. EKG shows left atrial hypertrophy. Chronic mitral stenosis would cause right ventricular hypertrophy and, thus, right ventricular heave secondary to pulmonary hypertension.

(Choice D) Hypokinesia of the inferior wall would be seen in case of inferior wall myocardial infarction, but clinical presentation of this patient is not suggestive of myocardial infarction.

(Choice E) Aortic valve vegetation can cause aortic regurgitation, which, if acute, would not have dilated heart; and, if chronic, would have eccentric hypertrophy of the heart. Also, patients with infective endocarditis appear sick, and they will have low to high-grade fevers, chills, or night sweats.

Page 30: T 20 Cardiology

(Choice F) Asymmetric septal hypertrophy is seen in hypertrophic cardiomyopathy. Hypertrophic cardiomyopathy is a disease of the young, and is less likely to present with acute cardiac failure, rather than sudden cardiac death which is commonly seen.

Educational Objective:Dilated cardiomyopathy may be seen following viral myocarditis (postpartum), particularly following Coxsackie B infection. Diagnosis is made by echocardiogram, which typically shows dilated ventricles and diffuse hypokinesia resulting in systolic dysfunction (low ejection fraction).

18% of people answered this question correctly

Test #20   Q.Id = 1857 Question 26 Of 26Item Review

Division : Cardiology

A 60-year-old man is brought to the ER by his wife because he lost consciousness in the bathroom at night. He says that he woke up, went to the bathroom to urinate, and fainted there. He rapidly recovered his consciousness without any indication of disorientation. He has never had such an episode before. He admits ‘problems with urination,’ including difficulty with initiating urination and frequent awakening to void at night. He does not take any medication. His past medical history is insignificant. He smokes 2 packs of cigarettes per day and does not consume alcohol. His blood pressure is 130/80 mm Hg while supine, and 132/80 while standing. His heart rate is 70/min. His physical examination is within normal limits. The ECG is normal. What is the most probable cause of the syncopal episode in this patient?

  A. Arrhythmia  B. Postural hypotension

C. Situational syncope  D. Seizure  E. Transient ischemic attack (TIA)

Explanation:

The most probable diagnosis in this patient is situational syncope related to micturition. The scenario described (middle age or older male with prostatic hypertrophy, who lost his consciousness after awakening and voiding at night) is typical for this type of syncope. The pathophysiologic mechanism underlying the situational syncope includes autonomic dysregulation, which can be partially explained by straining and rapid bladder emptying. Cardioinhibitory and/or vasodepressor mechanisms may be involved.

Cardiovascular causes like arrhythmia (Choice A) are less likely because there are no signs of a structural heart disease on the physical examination and

Page 31: T 20 Cardiology

the ECG.

Postural hypotension (Choice B) is unlikely because there are no signs of orthostatic BP changes, as well as no obvious cause for the problem, such as medications or hypovolemia.

A patient with no previous history and a rapid recovery of consciousness argue against seizure (Choice D) as a cause of the episode.

Transient ischemic attack (Choice E) is a rare cause of syncope, and is usually accompanied by symptoms of focal neurologic dysfunction (ataxia, paresis, diplopia).

Educational objective:Situational syncope should be considered in the differential diagnosis of syncopal episodes. The typical scenario would include a middle age or older male, who loses his consciousness immediately after urination, or a man who loses his consciousness during coughing fits.

26% of people answered this question correctly

Test #20   Q.Id = 593 Question 27 Of 26Item Review

Division : Cardiology

A 60-year-old female presents with dyspnea and ankle edema. Her dyspnea was mild about 6 months ago and used to occur with moderate exertion but it has now progressed to dyspnea even at rest. She denies any chest pain, syncope, or palpitations. She does not smoke or drink alcohol. She is not suffering from diabetes mellitus, hypertension or hyperlipidemia. Her vitals are, PR: 80/min; BP: 130/70 mm of Hg; Temperature: 37.1C(98.9F); RR: 14/min. Her examination is significant for jugular venous distension, bilateral ankle edema, and tender hepatomegaly. Chest auscultation shows bibasilar crackles. Heart sounds are distant and there is no murmur. Chest x-ray shows mild cardiomegaly and a right-sided pleural effusion. EKG shows low voltage QRS complexes and nonspecific ST-T wave changes. Echocardiography shows symmetrical thickening of the left ventricular wall. A diagnosis of restrictive cardiomyopathy is being considered. Which of the following abnormalities is most likely to be present in cases of restrictive cardiomyopathy?

  A. Severe systolic dysfunctionB. Severe diastolic dysfunction

  C. Supernormal ejection fraction  D. Dynamic outflow obstruction  E. Increased left ventricular volume

Explanation:

Page 32: T 20 Cardiology

Severe diastolic dysfunction is present in restrictive cardiomyopathy due to rigid and stiff ventricle wall. Systolic function is preserved in this condition.

Severe systolic dysfunction and increased left ventricle size are features of dilated cardiomyopathy.

Supernormal ejection fraction is present in hypertrophic cardiomyopathy as is dynamic outflow obstruction. Diastolic dysfunction is present in HCM, due to the stiff, hypertrophied ventricle wall.

Educational Objective: Understand the presentation of restrictive cardiomyopathy and know its pathophysiology and hemodynamics

56% of people answered this question correctly

Test #20   Q.Id = 587 Question 28 Of 26Item Review

Division : Cardiology  

A 27-year-old black male has a history of repeated attacks of dyspnea upon exertion. He does not smoke, nor does he drink alcohol. He denies chest pain or syncope. His vitals are stable and he is afebrile. On examination, there is no edema or jugular venous distension. His lungs are clear to auscultation. Auscultation of precordium reveals a III/VI harsh diamond shaped systolic murmur best heard at the left lower sternal border. EKG shows evidence of left ventricular hypertrophy. Echocardiography is performed which shows asymmetrical septal hypertrophy and systolic anterior motion of the mitral valve. Which of the following is true regarding the inheritance of this disease?

A. Autosomal dominant  B. Autosomal recessive  C. X-linked recessive  D. X-linked dominant  E. Multifactorial

Explanation:

This patient is most likely suffering from hypertrophic cardiomyopathy that has an autosomal dominant mode of inheritance. More than 100 mutations have been identified and more than 40% involve cardiac beta-myosin heavy chain gene on chromosome 14. It has been found that about one third of the first-degree relatives of the patients of familial HCM have evidence of HCM. There are many sporadic cases of HCM that occur due to spontaneous mutations. It has been found that about one third of

 

Page 33: T 20 Cardiology

the first-degree relatives of the patients of familial HCM have evidence of HCM.

Educational Objective: HCM follows autosomal dominant inheritance.

42% of people answered this question correctly

Test #20   Q.Id = 605 Question 29 Of 26Item Review

Division : Cardiology  

A 65-year-old white male was diagnosed with congestive heart failure one year ago. His ejection fraction (EF) was estimated at 30-35% (Normal is 55-65%). His heart failure is secondary to chronic alcohol use. His other medical problems include type II DM that is being treated with insulin. His other medications include lisinopril, aspirin, and simvastatin. He had been doing well on these medications, until 5 days ago, when he started noticing palpitations, increasing dyspnea even at rest, and swelling of his ankles. He is today in your office for these symptoms. His PR: 122/min irregularly irregular; BP: 100/70mm Hg; Temperature: 37C(98.6F). EKG shows new onset atrial fibrillation. You increase his lasix dose. Which of the following is the most appropriate next step in the management of this patient's congestive heart failure?

A. Start digoxin  B. Start metoprolol  C. Start spironolactone  D. Start verapamil  E. Give IV dobutamine

Explanation:

The most appropriate next pharmacological measure for the treatment of this patient's CHF is the addition of digoxin to his regimen. Digoxin slows the ventricular response rate in atrial fibrillation by slowing down AV conduction and thus increasing the diastolic filling. It also has a beneficial effect in patients of systolic dysfunction due to its positive inotropic effect. Thus any patient who has atrial fibrillation and heart failure will benefit from digoxin; it should be considered over before beta-blockers or calcium channel blockers.

Another next best choice in the management of this patient' which is not given in the choices would be to anticoagulate the patient, as this is one of the most important interventions in reducing the morbidity and mortality associated with atrial fibrillation

Verapamil, as well as metoprolol, will slow down the ventricular response rate by its negative dromotropic effect but it may worsen systolic dysfunction by its

Page 34: T 20 Cardiology

negative inotropic effect. Beta-blockers are indicated for heart failure patients who are NYHA class II or III with systolic dysfunction. Beta-blockers are preferred over the digoxin or calcium channel blockers in patients with coronary artery disease. Calcium channel blockers are not the best drugs for patients with heart failure due to their negative inotropic effect.

Furosemide, a loop diuretic, should be given in increased doses at this stage to remove the excess of fluid responsible for the congestive heart failure symptoms in this patient. However, this is not going to slow down the ventricular response rate in the setting of AF that is the main cause of decompensation in this patient. Adding spironolactone is not going to provide enough diuresis at this time but it may be useful chronically in patients with severe CHF by inhibition of aldosterone pathways. Indeed some studies have shown an improvement in mortality with the use of spironolactone in heart failure patients.

IV dobutamine is used when a patient is in cardiogenic shock. This patient's heart rate is already high (dopamine causes tachycardia) and his BP is not very low. So, using dopamine is not indicated.

Educational Objective: Know how to manage a case of CHF exacerbation due to atrial fibrillation with a rapid ventricular response. Digoxin is the drug of first choice in this situation.

42% of people answered this question correctly

Test #20   Q.Id = 1107 Question 30 Of 26Item Review

Division : Cardiology

A 24-year-old man rushes into the ER complaining of severe chest pain. He says he was fine until 30 minutes prior, when he developed sudden onset of chest pain. He denies any palpitations, shortness of breath, cough or fever. He denies any previous episodes of chest pain. Past medical history is unremarkable except for an appendectomy done one year back. He denies the use of any prescription or over the counter medications. He denies any illicit drug use, but admits to alcohol intake. His father died at the age of 64 due to “some heart problem” and his mother died of ovarian cancer. The patient is agitated and is sweating profusely. His vital signs are as follows: PR: 104/min, B.P: 140/90 mm Hg, and R.R: 14/min. Physical examination is normal except for dilated pupils and a small amount of blood at the external nares. An EKG done shows ST elevation in leads V1-V4. The most likely explanation of his symptoms would be?

  A. Myocardial infarction from atherosclerotic vascular disease  B. Acute pericarditis from Coxsackie virus  C. Pneumonitis from Streptococcus pneumoniae

D. Drug induced vasospasm

Page 35: T 20 Cardiology

  E. Aortic dissection  F. Drug induced pulmonary infarction

Explanation:

Patients with cocaine intoxication usually present with anxiety, aggressiveness, agitation, and psychosis or delirium in the setting of a recent cocaine use. In addition, they could develop elevated or low blood pressure, tachycardia or bradycardia, profuse sweating, pupillary dilatation, nausea or vomiting, and insomnia. Overdoses can be fatal as these patients can develop cardiac arrhythmias, myocardial infarcts, seizures or stroke. Not very frequently, these patients present with formication also known as “cocaine bugs”, where the patient thinks that there are bugs crawling all over him. Nosebleeds can occur in patients who snort cocaine. Although a more complete workup would be necessary before making a diagnosis, this question asks for the MOST LIKELY diagnosis. This patient denies any illicit drug use, but it is common to find patients who deny drug use and have positive urine toxicology screens.

Although this patient does have ST elevation in the anterior leads, considering his age, and other features like pupillary dilatation and bleeding from the nose, cocaine use seems more likely. However, a complete cardiac work up is mandatory to rule out other cardiac causes for his symptoms. A chest x-ray, cardiac enzymes and urine toxicology screen would be necessary to make an accurate diagnosis.

For the same reasons as above, cocaine intoxication would be the most likely diagnosis. Acute pericarditis would present with diffuse ST elevations (ST elevations in all the leads) on EKG.

In the absence of cough, fever and shortness of breath, Pneumonitis would be unlikely in this patient. Besides, with his EKG findings, pneumonitis can clearly be ruled out.

Aortic dissection usually does not cause pupillary dilatation; however, those patients may have severe chest pain and hypertension,

Drug induced pulmonary infarction would not cause ST elevation in V1 to V4.

Educational Objective: Think of cocaine intoxication in a young patient presenting with chest pain/myocardial infarction or stroke. Features of cocaine intoxication are cocaine bugs, agitation, decreased appetite, dilated pupils, elevated or decreased blood pressure, tachycardia or bradycardia, and sweating.

42% of people answered this question correctly

Test #20   Q.Id = 1569 Question 31 Of 26 Division : Cardiology  

Page 36: T 20 Cardiology

Item Review

A 45-year-old recently migrated Mexican farmer comes to your office because of dyspnea and fatigue, since last 2 months. His vital signs are, BP: 126/80 mm Hg, PR: 80/min, RR: 16/min and Temperature: 37C(98F). On examination, he has pedal edema, elevated jugular venous pressure with positive Kussmaul’s sign and increased abdominal girth with free fluid. Auscultation reveals reduced intensity of apex beat with an early heart sound following S2. Jugular venous pressure tracing shows prominent ‘x' and ‘y' descent. Which of the following is the most likely cause for this patient’s symptoms?

  A. Cor pulmonaleB. Tuberculosis

  C. Viral infection  D. Psittacosis  E. Pneumoconiosis

Explanation:

The diagnosis in this patient is constrictive pericarditis. Constrictive pericarditis results from obliteration of pericardial space and fibrosis of pericardium following an acute fibrinous or serofibrinous pericarditis or chronic pericardial effusion. Tuberculosis is a very common cause of constrictive pericarditis, especially in developing countries. This patient is a recently migrated Mexican farmer, which puts TB on the top of the list of differentials for his constrictive pericarditis (Option E). Some other causes of constrictive pericarditis are idiopathic (42% in USA), post radiotherapy (31% in USA), post surgical (11%), connective tissue disorders, neoplasm, uremia, sarcoidosis, etc.

Constrictive pericarditis will lead to inability of ventricle to fill properly during diastole and would further cause the signs and symptoms of decreased cardiac output (fatigue, muscle wasting etc) and /or signs and symptoms of venous overload like elevated JVP, dyspnea, ascites, positive Kussmaul's sign, pedal edema, tender hepatomegaly etc. Sharp 'x' and 'y' descent on central venous tracing is characteristic of constrictive pericarditis as is the presence of pericardial knock (early heart sound heard after S2).

Option D and E: The clinical presentation of this patient is highly suggestive of constrictive pericarditis and psittacosis and pneumoconiosis are not an etiological cause of this condition.

Option A: This patient has no finding suggestive of chronic lung disease, which would lead to cor pulmonale. Also cor pulmonale would not cause constrictive pericarditis.

Educational objective: Tuberculosis is the most common cause of constrictive pericarditis, in immigrant

Page 37: T 20 Cardiology

population. It should be considered in patients with unexplained elevation of JVP and history of predisposing condition.

16% of people answered this question correctly

Test #20   Q.Id = 1701 Question 32 Of 26Item Review

Division : Cardiology

45-year-old male presents to your clinic with complaints of pleuritic chest pain. He says he is a heavy smoker and has been having intermittent cough for the past 2 weeks. His blood work is normal and his chest-x ray is clear. EKG reveals a normal sinus rhythm with a heart rate of 68. He does have a PR interval > 0.2 seconds. The next step in the management of his arrhythmia is:

A. Observation  B. Atenolol  C. Pacemaker  D. Electrophysiology study  E. Cardiac catheterization  F. Atropine

Explanation:

First-degree heart block is characterized by a prolonged constant PR interval (>0.2s). The heart block may be entirely normal or may be secondary to increased vagal tone or use of digitalis. The AV conduction must be normal and intact. When first-degree heart block is identified, no treatment is required. The majority of patients are asymptomatic and remain so. Almost all cases of first-degree heart block are discovered as an incidental finding while working up the patient for a different disorder.

Option A: First-degree heart block requires no treatment. The heart rate is not affected and neither bradycardia nor any form of AV block is seen. Atropine, which is an anticholinergic agent, is never used to treat first-degree heart block. Atropine increases heart rate.

Option B: Beta-blockers have no role in the treatment of heart block of any form. Beta-blockers can precipitate and worsen heart block. First-degree heart block does not affect the heart rate or the force of contraction. Beta-blockers are generally used as anti-hypertensives, anti anginals and anti arrhythmics.

Option C: Pacemakers are never inserted for first-degree heart block. Pacemakers are more suitable for bradycardia or trifascicular blocks.

Option D: When an individual is diagnosed with a first-degree heart block, no

Page 38: T 20 Cardiology

further studies are required. The disorder is benign and does not progress to heart block of any form. If the patient has been on digoxin, digoxin levels may be ordered or the drug with held for a few days. Electrophysiological studies are generally done when the site and focus of arrhythmias is required for ablation or resection. EP studies are almost never done for first-degree heart block, as there is no pathology ever identified.

Option E: Catheterization is an invasive procedure and certainly not indicated for this benign arhythmia.

Option F: Atropine is used for symptomatic bradycardia resulting from type II and type III heart block, not for type I block.

Educational objective:First-degree heart block is a completely benign arrhythmia and requires no treatment.

41% of people answered this question correctly

Test #20   Q.Id = 915 Question 33 Of 26Item Review

Division : Cardiology  

A 45-year-old white male is brought to the emergency room with sudden onset of substernal chest pain. Pain is 6-8/10 in severity, it is alleviated by leaning forward and worsened by coughing and deep breathing; he says the pain goes to his left arm. He had an upper respiratory tract infection 1 week ago. He is a non-smoker and does not drink alcohol. He denies any cough or dyspnea. His vitals are stable except for a mild fever. Chest examination is significant for a pericardial friction rub. Chest-X ray is normal and EKG shows diffuse ST segment elevation with ‘J’ point elevation, no ‘Q’ waves, and no reciprocal changes. Which of the followings is the most likely cause of his symptoms?

A. Coxsackie virus  B. Mycobacterium tuberculosis  C. Borrelia burgdorferi  D. Uremia  E. Streptococcal pneumoniae  F. Coronary artery disease

Explanation:

Severe constant pain that localizes over the anterior chest, radiates to the arm, shoulder, back, epigastrium or neck, is intensified with inspiration, and relieved by sitting up and leaning forward represents a classic presentation of acute pericarditis. The pain is often difficult to differentiate from an ischemic pain because often it is very severe, short, and sometimes has a constricting quality that radiates either to one or both arms. However, pain which is relieved by

Page 39: T 20 Cardiology

sitting up and leaning forward and is intensified by inspiration or lying supine is characteristic of acute pericarditis. Acute pericarditis has a very broad differential diagnosis. But it is most commonly caused by viruses and Coxsackie virus is a frequent offender. A history of a recent viral illness is common. Other viruses implicated include echovirus, adenovirus, EBV, and HIV. The classic findings of pericarditis on EKG are:1.Diffuse ST segment elevation with upward concavity at ‘J’ point. 2.No new ‘Q’ waves 3.'PR’ segment elevation in aVR with ‘PR’ depression in other leads.

Mycobacterium tuberculosis is an important cause of pericarditis but it is very rare in the absence of other risk factors thus highly unlikely in this patient.

Uremia can cause acute pericarditis and this may represent an indication for dialysis. The above patient does not have any clue about renal insufficiency.

The above patient does not have pneumonia.

B Burgdorferi, the agent of Lyme disease, may cause an acute myopericarditis but is not likely to cause isolated acute pericarditis. Cardiac involvement in Lyme disease occurs in 8% of patients with Lyme disease and usually the most common type of it is fluctuating degrees of atrioventricular block.

Educational Objective:Recognize the clinical presentation of acute pericarditis. Coxsackie-B virus is a common cause of acute pericarditis.

57% of people answered this question correctly

Test #20   Q.Id = 31 Question 34 Of 26Item Review

Division : Cardiology

A 65-year-old white female, with chronic congestive heart failure, was admitted for community-acquired pneumonia. She is receiving aspirin, digoxin, furosemide, levofloxacin, and oral simvastatin. She is feeling better and her pneumonia is resolving. While planning for discharge she developed recurrent sustained ventricular tachycardia with stable vital signs during the nighttime. The patient has recieved amiodarone and now is stable in sinus rhythm. A recent echocardiogram showed an ejection fraction (EF) of 35%. What is the most important next step in the management at this stage?

  A. Add beta blocker  B. Add spiranolactone

C. Measure serum electrolytes  D. Stop simvastatin  E. Discharge the patient

Page 40: T 20 Cardiology

Explanation:

When someone is having recurrent VT, first thing to do after stabilizing the patient is to search for underlying cause. This patient, most likely, has an electrolyte imbalance due to diuretics. Furosemide commonly causes hypokalemia, which may lead to digoxin toxicity. Therefore, ordering serum electrolytes and serum digoxin level is the most reasonable approach.

Adding a beta-blocker is a good thing for chronic heart failure but not the next step in this case.

Simvastatin has nothing to do with ventricular tachycardia.

Spironolactone is a potassium sparing diuretic that is not indicated at this stage.

Educational Objective: Look for electrolyte abnormalities and correct them in patients with arrhythmias.

31% of people answered this question correctly

Test #20   Q.Id = 644 Question 35 Of 26Item Review

Division : Cardiology  

A 43-year-old white male is found to have atrial premature complexes (APCs) on a routine EKG. The patient denies any chest pain, shortness of breath, or syncopal episodes. He has been smoking cigarettes 1-2 packs/day for the past 20 years. He has been an alcoholic for 20 years and still drinks 1-2 beers/day. His family history is significant for his mother having had an MI at the age of 65 and his father having had a stroke at the age of 72. He has no personal history of hypertension or diabetes. A complete physical examination is unremarkable. His vitals are BP: 130/70 mm Hg; PR: 80/min and afebrile. Which of the following interventions is the most appropriate regarding the management of this patient?

  A. Reassurance, and send the patient home  B. Place the patient on 24-hour Holter monitor  C. Do an echocardiogram  D. Start a beta blocker

E. Advise him to stop alcohol and tobaccoExplanation:

An atrial premature beat results from the premature activation of the atria that originates from a site other than the SA node. EKG shows an early P wave. Atrial premature beats may be present with underlying heart disease or may even be present in the absence of any underlying heart disease. They are usually asymptomatic or can cause palpitations. They may be single or may be seen in the pattern of bigeminy. Occasionally they may result in supraventricular

Page 41: T 20 Cardiology

or less commonly ventricular arrhythmias.

Treatment is required only when the patient with atrial premature contraction has disturbing palpitations or supraventricular tachycardia. Precipitating factors like tobacco, alcohol, or adrenergic agonists need to be identified and corrected before opting for pharmacological therapy with beta-blockers or calcium channel blockers. The above patient has precipitating factors that need to be corrected. He is asymptomatic and does not have supraventricular tachycardia; therefore there is no need for pharmacological treatment.

Reassurance alone is not appropriate as the identified risk factors for the development of atrial premature contractions need to be removed.

This patient has no valvular, coronary artery disease or heart failure and echocardiography is therefore not required.

Holter monitoring is used both for diagnostic and prognostic purposes. In the above case, diagnosis is already established and his arrhythmia so far is benign. Therefore, there is no need for Holter monitoring.

Educational Objective: Tobacco and alcohol are reversible risk factors for the development of atrial premature beats.

43% of people answered this question correctly

Test #20   Q.Id = 980 Question 36 Of 26Item Review

Division : Cardiology  

A 34-year-old Caucasian male, who has a 10 year history of asymptomatic Wolff-Parkinson-White syndrome, presented with 4 hours history of palpitations. He denies any chest pain, shortness of breath, fevers, or chills. On examination, he has an irregularly irregular pulse. EKG shows atrial fibrillation with a rate of 110/min associated with Wolff-Parkinson-White syndrome. His BP: 120/80 mmHg; RR: 18/min; Temperature: 37C(98.6F). Which of the following interventions is the next most appropriate in the management of this patient?

  A. IV digoxin.  B. IV verapamil.  C. IV lidocaine.

D. IV procainamide.  E. Cardioversion.

Explanation:

The drugs that slow the AV conduction (digoxin and verapamil) are

Page 42: T 20 Cardiology

contraindicated in patients with atrial fibrillation and Wolff-Parkinson-White syndrome because they can increase the conduction of impulses through the accessory pathway, thus leading to malignant arrhythmias and hypotension.

Lidocaine, as well in some cases, may worsen the situation by increasing the conduction through the AV node. That is why lidocaine is usually not a preferable drug in this situation.

Cardioversion is usually indicated in patients who are hemodynamically unstable (very rapid ventricular rates with hypotension). In those circumstances the likely chances of conversion to normal sinus rhythm with drug therapy is very low. However, this patient is relatively stable with a normal blood pressure so these patients are more likely to respond with IV procainamide and it is the drug of choice in these circumstances. You can also use IV disopyramide.

Educational Objective:Procainamide or disopyramide are the drugs of choice for Atrial fibrillation in the context of WPW syndrome. Remember digoxin and calcium channel blockers should not be used.

17% of people answered this question correctly

Test #20   Q.Id = 2037 Question 37 Of 26Item Review

Division : Cardiology  

A 20-year-old female presents to your office for a routine check-up. She has no complaints and her past medical history is insignificant. She is not taking any medications and denies drug abuse. Her blood pressure is 125/65 mmHg and heart rate is 80/min. Cardiac auscultation reveals early diastolic murmur at the left sternal border. What is the next best step in the management of this patient?

  A. Electrocardiogram  B. Chest x-ray  C. Coronary angiography

D. Echocardiography  E. No further work-up

Explanation:

Diastolic and continuous murmurs revealed on cardiac auscultation should always be investigated, because the probability that an organic cause is present is high. Midsystolic soft murmurs (grade I-II/IV) in an asymptomatic young patient are usually benign and need no further work-up (Choice E). Transthoracic Doppler echocardiography is a non-invasive and very efficient tool to confirm the presence of flow abnormalities, find the cause, and assess the

Page 43: T 20 Cardiology

extent of the problem.

Catheterization and angiography (Choice C) can also reveal the structural defect and assess the severity of the process, but it is invasive; therefore, echocardiography is employed first.

Chest x-ray (Choice B) and electrocardiogram (Choice A) are too non-specific and have limited use in such a case.

Educational Objective:Diastolic and continuous murmurs as well as loud systolic murmurs revealed on cardiac auscultation should always be investigated using transthoracic Doppler echocardiography. Midsystolic soft murmurs (grade I-II/IV) in an asymptomatic young patient are usually benign and need no further work-up.

33% of people answered this question correctly

Test #20   Q.Id = 2029 Question 38 Of 26Item Review

Division : Cardiology  

A 47-year-old male was brought to the emergency room with chest pain of acute onset. The pain was associated with nausea, vomiting, and diaphoresis. He has a history of diabetes, hypertension, and hyperlipidemia. ECG reveals ST segment elevation in the anterolateral leads and ventricular premature beats (VPBs). The patient dies within the first hour after the arrival to emergency room. What is the most likely pathophysiologic mechanism responsible for this patient’s death?

  A. Electro-mechanic dissociationB. Reentry

  C. Full conduction block  D. Increased automaticity  E. Asystole

Explanation:

The most common cause of death in patients with acute myocardial infarction (MI) is complex ventricular arrhythmia. Acute ischemia creates heterogeneity of conduction in the myocardium. Areas of partial block of conduction are frequently formed that predispose the patient to reentrant arrhythmia. Ventricular fibrillation is a typical example of reentrant arrhythmia. Decline in mortality of patients hospitalized with acute coronary syndromes is largely attributable to the effective detection and treatment of reentrant ventricular arrhythmias.

Full conduction block (Choice C) may occur in patients with acute myocardial

Page 44: T 20 Cardiology

infarction, especially inferior wall MI, but it is a less common cause of death.

Asystole (Choice E) usually occurs in patients with severe complications of MI (e.g., heart failure).

Electromechanical dissociation (Choice A) is typical for pulmonary thromboembolism and pericardial tamponade.

Increased automaticity (Choice D) is a frequent cause of arrhythmia in patients with glycoside intoxication.

Educational Objective:Reentrant ventricular arrhythmia (ventricular fibrillation) is the most common cause of death in patients with acute myocardial infarction.

4% of people answered this question correctly

Test #20   Q.Id = 24 Question 39 Of 26Item Review

Division : Cardiology  

A 63-year-old Caucasian male is admitted for sudden onset of severe chest pain. His ECG revealed ST elevation in leads V2-V6. He was given thrombolytic therapy, heparin, aspirin, metoprolol, morphine, and nitrates. His coronary angiogram, after thrombolytic therapy, revealed a 50% obstruction in the left anterior descending artery. On the 3rd day of his hospitalization the patient suddenly developed severe shortness of breath at rest and quickly became hypotensive. Examination reveals a soft S1, an apical pansystolic murmur (PSM) radiating to the axilla, and bibasilar crackles. His vitals are BP: 90/60 mm of Hg; HR: 102/min; RR: 30/min; Temperature: 37.8C (100F). An echocardiogram performed on the 2nd hospital day revealed an anterior akinetic segment. What is the most likely explanation for this patient's deterioration?

  A. Pericardial tamponade  B. Pulmonary embolism  C. Rupture of ventricular septum

D. Papillary muscle dysfunction  E. Acute aortic dissection

Explanation:

This patient has classic features of new onset mitral regurgitation. Sudden onset of shortness of breath (SOB), bibasilar rales, and an 'apical' murmur radiating to the axilla are quite characteristic of new onset mitral regurgitation. Papillary muscle dysfunction, or rupture, is the most common cause of MR in this setting.

Ventricular septal rupture has similar features but the murmur is heard at the left

Page 45: T 20 Cardiology

sternal border and would not radiate to the axilla.

Pericardial tamponade is a very important differential diagnosis in this patient but usually patients will not have any murmurs.

Pulmonary embolism (PE) and aortic dissection are completely different from this presentation. Pulmonary edema is not seen in PE.

Educational Objective: Recognize the early complications of an acute anterior wall MI.

50% of people answered this question correctly

Test #20   Q.Id = 619 Question 40 Of 26Item Review

Division : Cardiology

A 56-year-old male, who is being treated with sildenafil for his sexual dysfunction, develops intermittent substernal chest pain that radiates to his jaw and left upper arm. He describes the pain as a sense of heaviness, is 7/10 in severity, is brought on by exertion, and is relieved by rest. His past medical history is insignificant other than his sexual dysfunction. He is a non-smoker and non-alcoholic. EKG performed while having chest pain in the emergency department shows T-wave inversion in leads II, III and aVF. Cardiac enzymes are normal. Which of the following is true regarding the treatment of his ischemic symptoms with nitrates?

A. Nitrates should not be given to him within 24 hours of the last dose of sildenafil  B. Nitrates should not be given to him within 12 hours of the last dose of sildenafil  C. The dose of nitrates should be reduced  D. Nitrates can be given safely without any change in the usual nitrate dose  E. The dose of nitrates should be increased

Explanation:

Nitrates are contraindicated when a patient is continuously or intermittently taking sildenafil (Viagra). In such settings nitrates may cause syncope, MI, or sudden death when a patient has an acute coronary syndrome. It is recommended not to use nitrates within 24 hours of the last dose of sildenafil. The reason for the dangerous interaction between nitrates and sildenafil is that both induce nitric oxide mediated vasodilatation.

Reduction of the nitrate dose would not protect against the dangerous interaction with sildenafil and is not the right choice here.

Within 12 hours of the last dose of sildenafil, considerable quantities of sildenafil could still be present in the circulation and thus it would dangerously interact with the nitrates.

Page 46: T 20 Cardiology

An increase in the nitrates dose, or giving nitrates at the same dose in patients who are taking sildenafil, is likely to produce substantial vasodilatation resulting in syncope, MI, or even sudden death.

Educational Objective:Nitrates are contraindicated when a patient is continuously or intermittently taking sildenafil (Viagra).

37% of people answered this question correctly

Test #20   Q.Id = 26 Question 41 Of 26Item Review

Division : Cardiology

A 57-year-old female with known bronchial asthma presents to your office for evaluation of her HTN for the 3rd time. Her last 2 visits revealed BP: 170/90 and 174/92 after 10 min of rest. Her vitals for this visit are: BP: 170/90 mm Hg, HR: 78/min, RR: 18/min, Temperature: 37.8C (100F). She denies headaches and any other complaints. She has been on beclomethasone and albuterol inhalation for her chronic persistent asthma. She denies smoking or alcohol intake. She has no allergies. Her family history is significant for myocardial infarction in both her mother and father. She tried regular exercise and low salt diet without much improvement. Based on this information, what step would you take next?

  A. Ask her to stop asthma medications  B. Prescribe propranolol  C. Prescribe enalapril

D. Prescribe hydrochlorothiazide  E. Do echocardiogram

Explanation:

The first line medication for HTN in the general population is either a thiazide diuretic or beta-blocker. Non-selective beta-blockers are contraindicated in asthmatics especially if they are steroid dependent or on continuous albuterol. Stopping the asthma medications is not a good choice here. Use of propranolol is also contraindicated in this patient.

ACE inhibitors, like enalapril, are the 1st line drugs for diabetics with hypertension. These are usually 2nd line drugs for the general population.

Usually an echocardiogram is not required in an asymptomatic patient, although she may have concentric hypertrophy.

Page 47: T 20 Cardiology

Therefore the best choice in this patient is hydrochlorothiazide.

Educational Objective: Choose the appropriate initial antihypertensive therapy in a patient with asthma. HCTZ is the initial drug of choice in patients with chronic persistent asthma.

38% of people answered this question correctly

Test #20   Q.Id = 2062 Question 42 Of 26Item Review

Division : Cardiology  

A 40-year-old female presents to the emergency room with palpitations and lightheadedness of acute onset. Also, she has experienced insomnia, fatigability, and weight loss lately. She does not smoke or consume alcohol. She is not taking any medications. Her blood pressure is 110/80 mmHg and heart rate is 120/min, irregular. Physical examination reveals lid lag and fine tremor of the outstretched hands. ECG shows atrial fibrillation with rapid ventricular response. What is the next best step in the management of this patient?

  A. DigoxinB. Propranolol

  C. Lidocaine  D. Quinidine   E. Immediate cardioversion

Explanation:

This patient presents with signs and symptoms suggestive of Grave’s disease - insomnia, fatigability, weight loss, lid lag, and tremor. Atrial fibrillation is a common complication of hyperthyroidism. As in other cardiovascular complications, atrial fibrillation in patients with hyperthyroidism is believed to be caused by increased sensitivity of beta-adrenoreceptors to sympathetic stimuli. The best initial choice for these stable patients is a beta-blocker like propranolol. It not only helps to control the rhythm in tachysystolic atrial fibrillation, but also diminishes other symptoms of hyperthyroidism.

Digoxin (Choice A) is not a good choice, because rapid ventricular response in hyperthyroidism-related atrial fibrillation is resistant to cardiac glycosides.

Quinidine (Choice D), when used without pretreatment with rhythm-controlling drugs, may even worsen tachysystoly. It is not commonly used today for chemical cardioversion.

Electrical cardioversion (Choice E) is effective in restoring the sinus rhythm; but, if the underlying condition (hyperthyroidism) is not addressed, atrial fibrillation

Page 48: T 20 Cardiology

will most probably recur. Emergent cardioversion is indicated if the arrhythmia precipitates dangerous complications (like hypotension, angina, or heart failure), or if the patient is hemodynamically unstable.

Lidocaine (Choice C) is useless in this situation, because it is not effective in atrial arrhythmias.

Educational Objective:In patients with hyperthyroidism-related tachysystolic atrial fibrillation, a beta-blocker is the drug of choice.

15% of people answered this question correctly

Test #20   Q.Id = 1839 Question 43 Of 26Item Review

Division : Cardiology  

A 63-year-old female presents to your office for a routine check-up. She has no present complaints. Her past medical history includes DM, type 2, and hypertension. Her current medications include glyburide and atenolol. She does not smoke. She drinks 2-3 glasses of wine 1-2 times a week. Three consecutive BP measurements were in the range of 138-142/87-90 mmHg. Physical examination is within normal limits. Her recent fasting glucose level was 250 mg/dL. ECG recorded 1 month ago showed left ventricular hypertrophy. Which statement about the BP control in this patient is the most accurate?

  A. BP is within acceptable range  B. BP is within optimal range

C. It is better to keep systolic pressure < 130 mmHg to slow end-organ damage  D. Diastolic BP is within acceptable range, but systolic is not  E. Systolic BP is within acceptable range, but diastolic is not

Explanation:

Traditionally a goal, blood pressure is considered to be below 140/90 mmHg. But it is recently recognized that BP needs more tight control in diabetics and patients with chronic renal failure. These two groups of patients are especially sensitive to high BP, that’s why the values of systolic BP for these patients should be kept lower than 130 mmHg and diastolic BP lower than 80 mmHg to prevent end organ damage.

Blood pressure could be considered in acceptable range (Choice A) if the patient was not diabetic. According to JNC-7 report (JNC stands for Joint National Committee on Prevention, Detection, Evaluation, and Treatment of High Blood Pressure) BP is considered optimal when it is below 120/80 mmHg (Choice B). In this patient both systolic (Choice E) and diastolic (Choice D) BP

Page 49: T 20 Cardiology

are above the goal values.

Educational objective:It is better to keep systolic pressure < 130 mmHg to slow end-organ damage in patients with diabetes and chronic renal failure.

24% of people answered this question correctly

Test #20   Q.Id = 1913 Question 44 Of 26Item Review

Division : Cardiology  

A 40-year-old African American male presents to the Emergency Room with a two-month history of occasional severe headache and blurring of vision. His past medical history is significant for hypertension controlled with hydrochlorothiazide for two years. His family history is significant for hypertension and diabetes. He smokes two packs a day and occasionally consumes alcohol. His blood pressure is 200/140 mmHg and heart rate is 75/min. Which of the following is most consistent with a diagnosis of malignant hypertension in this patient?

  A. Left ventricular hypertrophy on ECG  B. Elevated serum creatinine level

C. Papilledema  D. Oliguria  E. Blood pressure >/= 200/140 mmHg

Explanation:

Ophthalmoscopy is an important component of the evaluation of patients with hypertension. Presence of papilledema in this patient is consistent with a diagnosis of malignant hypertension. Although the blood pressure of the patients with malignant hypertension is usually >/= 200/140 mmHg (Choice E), papilledema should be present in order to make a diagnosis of malignant hypertension. Renal failure with elevated creatinine level (Choice B) and oliguria (Choice D) can develop rapidly in this case without treatment, but its presence is not required to diagnose malignant hypertension. Left ventricular hypertrophy on ECG (Choice A) is not a diagnostic criterion for malignant hypertension.

Educational Objective:Always suspect malignant hypertension in patients with very high blood pressure (>/= 200/140 mmHg). Presence of papilledema on ophthalmoscopy confirms the diagnosis. The pathologic change responsible for end-organ damage in malignant hypertension is fibrinoid necrosis of small arterioles.

Page 50: T 20 Cardiology

17% of people answered this question correctly

Test #20   Q.Id = 608 Question 45 Of 26Item Review

Division : Cardiology

56-year-old white male presents to the emergency room with central chest pain of sudden onset that woke him up from sleep. Pain is 9/10 in severity, feels like a pressure sensation, radiates to left upper arm and lower jaw, and is not relieved by sublingual nitroglycerin. He has 40 pack year history of smoking and he is an occasional alcohol drinker. His HTN was diagnosed 2 years ago and he is not taking any medication for it. He is anxious and sweating profusely. His lungs are clear to auscultation. Acute MI is a strong consideration on your differential diagnosis. Which of the following would you expect to be the earliest EKG finding in a patient with an acute MI?

A. Peaked 'T' wave  B. 'ST'segment elevation  C. 'T' wave inversion  D. Appearance of 'Q' wave  E. 'ST' depression with 'T' inversion

Explanation:

The earliest EKG finding in acute MI is peaked (hyperacute) T waves, followed by ST segment elevation, followed by the inversion of T waves, followed by the appearance of Q waves.

Q waves do not occur in 30-50% cases of acute infarcts (Non Q wave MI).

The earliest changes of hyperacute ‘T’ waves are frequently not seen in clinical practice because by the time the patient present they already have ‘ST’ elevation.

Educational Objective: Know the progression and timeframe of the ECG findings in the setting of an acute MI. The earliest EKG finding in acute MI is peaked (hyperacute) T waves.

17% of people answered this question correctly

Test #20   Q.Id = 601 Question 46 Of 26Item Review

Division : Cardiology

A 60-year-old woman presents to the emergency room for the acute onset of breathlessness. She has had similar episodes on three prior occasions over the last few months. Her past medical history is significant for rheumatic fever as a child. Her vitals are, PR: 115/min and irregularly irregular; BP: 130/70 mm of Hg; Temperature: 37.1C(98.7F); RR: 14/min. Her apex beat is tapping in quality and

Page 51: T 20 Cardiology

is in left fifth intercostal space just inside the mid-clavicular line. Cardiac auscultation shows a loud first heart sound and a mid-diastolic rumble best heard at the apex. Auscultation of her lungs reveals crepitations in both lung fields. Based on these findings, what is the most likely diagnosis of this patient?

A. Mitral stenosis  B. Mitral regurgitation  C. Tricuspid stenosis  D. Aortic stenosis  E. Aortic insufficiency

Explanation:

This patient most likely suffers from mitral stenosis, which is the most frequent valvular disease caused by rheumatic heart disease. The presence of a tapping apex beat, a loud first heart sound, and a mid-diastolic rumble at the apex suggest the diagnosis of mitral stenosis in this patient. Atrial fibrillation is quite common in patients with mitral stenosis.

Mitral regurgitation may also occur in rheumatic heart disease but it is characterized by a pansystolic murmur best heard at the apex that radiates to the axilla.

Aortic stenosis, aortic insufficiency, and tricuspid stenosis occur less frequently in the setting of rheumatic heart disease and the auscultatory findings in this patient are not compatible with any of these disorders.

The murmur of aortic regurgitation occurs in early diastole, is decrescendo, with a high-pitched, blowing quality and is best heard in the left third intercostal space. The murmur intensifies by leaning forward and holding the breath in expiration.

The murmur of aortic stenosis occurs in systole (systolic ejection murmur), and radiates to the neck.

The murmur of tricuspid stenosis occurs in mid to late diastole (presystolic), it increases in inspiration, and is better heard at the left lower sternal border or in the epigastrium. When atrial fibrillation is present, a mid-diastolic rumble can be heard.

Educational Objective: Recognize the clinical signs of mitral stenosis, and know how to differentiate it clinically from other valvulopathies.

64% of people answered this question correctly